Đến nội dung

toanND nội dung

Có 50 mục bởi toanND (Tìm giới hạn từ 25-05-2020)



Sắp theo                Sắp xếp  

#719140 toán rời rạc

Đã gửi bởi toanND on 06-01-2019 - 16:14 trong Tổ hợp và rời rạc

Cho tập hợp X = {1;2;...;2019}. Chứng minh rằng với ba phần tử khác nhau thuộc X luôn tìm được trong chúng hai số a, b mà $|\sqrt[5]{a}-\sqrt[5]{b}|=1$ với $(\sqrt[5]{x})=x$$, \forall x\in \mathbb{R}$




#719411 $a+b+c\geqslant ab+bc+ca$

Đã gửi bởi toanND on 12-01-2019 - 23:02 trong Bất đẳng thức - Cực trị

Cho 3 số thực không âm a, b, c thỏa mãn $\frac{1}{a+b+1}+\frac{1}{b+c+1}+\frac{1}{c+a+1}\geqslant 1$. Chứng minh rằng :

$a+b+c\geqslant ab+bc+ca$




#719725 $S = \frac{ab}{c}+\frac{bc}...

Đã gửi bởi toanND on 26-01-2019 - 17:37 trong Bất đẳng thức - Cực trị

Cho a, b, c thỏa mãn a2 + b+ c2 = 1. Tìm GTNN của biểu thức $S = \frac{ab}{c}+\frac{bc}{a}+\frac{ca}{b}$




#719872 $C,F,M,N$ đồng viên

Đã gửi bởi toanND on 02-02-2019 - 11:43 trong Hình học

Cho hai đường tròn $(\omega _{1})$ và $(\omega _{2})$ cắt nhau tại hai điểm A và B. Một đường thẳng qua B cắt $(\omega _{1})$ và $(\omega _{2})$ tại C và D. Một đường thẳng khác qua B cắt $(\omega _{1})$, $(\omega _{2})$ tại P, Q. Chứng minh rằng nếu CD = EF thì C, F, M, N đồng viên với M, N lần lượt là điểm chính giữa các cung nhỏ BP và BQ.




#720157 Cho tứ giác lồi $ABCD...,$ tính $BC$

Đã gửi bởi toanND on 13-02-2019 - 22:08 trong Hình học

Cho tứ giác lồi $ABCD$ có $AD=\sqrt{3},\widehat{ABD}=\widehat{ACD}=60^{0},E$ và $F$ là tâm đường tròn nội tiếp các tam giác $ABD,ACD.$

Biết $EF=\frac{\sqrt{3}-1}{2},$ tính $BC.$




#720250 $M$ là trung điểm $PQ$

Đã gửi bởi toanND on 17-02-2019 - 11:05 trong Hình học

Cho tam giác ABC, M là trung điểm BC. Gọi (I) là đường tròn nội tiếp tam giác ABC. Giả sử AM cắt (I) tại K, L (AK > AL). Các đường thẳng song song với BC qua K, L cắt (I) lần lượt tại X, Y. AX, AY cắt BC lần lượt tại P, Q. Chứng minh rằng: M là trung điểm PQ

Capture 2.PNG




#720265 $ii) f(xy)=f(x).f(y)-f(x+y)+1$

Đã gửi bởi toanND on 17-02-2019 - 18:19 trong Phương trình hàm

Tìm hàm số $f:\mathbb{Q}\rightarrow \mathbb{Q}$ thỏa mãn:

$i) f(1)=2$

$ii) f(xy)=f(x).f(y)-f(x+y)+1$




#720532 $2(x^{2}-4x+6)-3\sqrt[3]{x^{3}+27}=0...

Đã gửi bởi toanND on 28-02-2019 - 20:49 trong Phương trình - Hệ phương trình - Bất phương trình

Giải phương trình sau : $2(x^{2}-4x+6)-3\sqrt[3]{x^{3}+27}=0$




#720596 $f(f(x)-y)=f(x^{2})+f(y)-2yf(x), \forall x,y\in...

Đã gửi bởi toanND on 02-03-2019 - 17:08 trong Phương trình hàm

Tìm tất cả các hàm $f:\mathbb{Z}\rightarrow \mathbb{Z}$ thỏa mãn các điều kiện:

i) $f(f(x)-y)=f(x^{2})+f(y)-2yf(x), \forall x,y\in \mathbb{Z}$

ii) $f(1)>0$




#721264 $\sum \frac{a+b}{c} \geq 2\sqrt{( \sum a)(...

Đã gửi bởi toanND on 03-04-2019 - 23:52 trong Bất đẳng thức - Cực trị

Cho các số thực dương a,b,c. Chứng minh rằng khi đó ta có:

$\frac{a+b}{c}+\frac{b+c}{a}+\frac{c+a}{b}\geqslant 2\sqrt{(a+b+c)(\frac{a}{bc}+\frac{b}{ca}+\frac{c}{ab})}$




#721410 Chứng minh rằng $T_{1}T_{2}, O_{1}O_{...

Đã gửi bởi toanND on 14-04-2019 - 10:00 trong Hình học

Cho tam giác ABC nội tiếp đường tròn (O). P là một điểm trên cạnh BC. Gọi (O1) là đường tròn tiếp xúc trong với (O) tại T1 và tiếp xúc với hai cạnh PA, PB. (O2) là đường tròn tiếp xúc trong với (O) tại T2 và tiếp xúc với hai cạnh PA, PC. Chứng minh rằng $T_{1}T_{2}, O_{1}O_{2}, BC$ đồng quy.




#721662 Chứng minh rằng trung điểm đoạn thẳng nối tâm

Đã gửi bởi toanND on 23-04-2019 - 21:33 trong Hình học

Cho tứ giác ABCD nội tiếp đường tròn (O) đường kính BD. E là giao điểm của AC và BD. Chứng minh rằng trung điểm đoạn thẳng nối tâm đường tròn ngoại tiếp tam giác AEB và tâm đường tròn ngoại tiếp tam giác BEC nằm trên BD. (sử dụng định lí con bướm)

capture 3.PNG




#722132 Chứng minh rằng số đo góc MPN luôn không đổi khi D thay đổi

Đã gửi bởi toanND on 11-05-2019 - 12:27 trong Hình học

Cho tam giác ABC, giả sử có điểm P nằm trong tam giác ABC sao cho góc BPC = góc CPA = góc APB. PB, PC theo thứ tự cắt CA, AB tại E, F. D là điểm di chuyển trên cạnh BC. Đường thẳng DF cắt đường thẳng AC tại M. Đường thẳng DE cắt đường thẳng AB tại N.

1. Chứng minh rằng số đo góc MPN luôn không đổi khi D thay đổi.

2. Gọi giao điểm của đường thẳng EF với đường thẳng MN là Q. Chứng minh rằng PQ là phân giác của góc MPN.

Capture 4.PNG




#722159 Chứng minh rằng số đo góc MPN luôn không đổi khi D thay đổi

Đã gửi bởi toanND on 12-05-2019 - 17:01 trong Hình học

Xét phép nghịch đảo tâm $P$ phương tích bất kì, ta đưa bài toán đã cho về bài toán mới sau.

Bài toán mới. Cho $\Delta ABC,P$ là điểm trong tam giác thoả $\widehat{APB}= \widehat{BPC}= \widehat{CPA}.PB,PC$ theo thứ tự cắt $(CPA),(APB)$ tại $E,F.D$ là điểm di chuyển trên $(PBC),(PDF)$ cắt $(PAC)$ tại $M,(PDE)$ cắt $(PAB)$ tại $N.$

a) Chứng minh số đo góc $\widehat{MPN}$ không đổi.

b) $(PEF)$ cắt $(PMN)$ tại $Q.$ Chứng minh $PQ$ là phân giác $\widehat{MPN}.$

Chứng minh.

a) Gọi $G,H,I,J,K,L,O$ là tâm các đường tròn $(PAC),(PAB),(PDF),(PDE),(PBC),(PEF),(PMN).$

Từ cách xác định các điểm này, ta có các bộ điểm thẳng hàng: $\overline{L,H,I}, \overline{I,K,J}, \overline{J,G,L}, \overline{I,O,G}, \overline{J,O,H}.$

Ta cũng có $IL \perp PC,JL \perp PB,GK \perp PC,HK \perp PB,HG \perp PA.$ Với chú ý $\widehat{APB}= \widehat{BPC}= \widehat{CPA}=120^0,$ ta được $\Delta KGH$ đều và $\widehat{KHI}= \widehat{KGI}=60^0.$

Do đó $IH \parallel GK,HK \parallel GJ \Rightarrow \Delta HIK \sim \Delta GKJ \Rightarrow \frac{HI}{HG}= \frac{HI}{HK}= \frac{GK}{GJ}= \frac{GH}{GJ} \Rightarrow \Delta IHG \sim \Delta HGJ$

$\Rightarrow \widehat{IOH}= \widehat{OGH}+ \widehat{OHG}= \widehat{OHG}+ \widehat{OJG}= 180^0- \widehat{HGJ}=60^0.$

Mà $MP \perp OI,NP \perp OH \Rightarrow \widehat{MPN}=60^0.$ Ta có đpcm.

b) Theo cách xác định các tâm đường tròn ở câu a), $\Delta HLG$ đều. Lại có $\widehat{IOH}=60^0$ theo câu a) nên $L$ là trung điểm cung $HG$ không chứa $O$ của $(HOG) \Rightarrow OL$ là phân giác $\widehat{HOG}.$

Lại có $MP \perp OG,NP \perp OH,PQ \perp OL \Rightarrow PQ$ là phân giác $\widehat{MPN}.$ Ta có đpcm.

 

có cách nào k sử dụng phép nghịch đảo k ạ? Tại e chưa học cái này




#722559 Dựng đường tròn $\omega _{1}$

Đã gửi bởi toanND on 27-05-2019 - 21:13 trong Hình học

Cho đường tròn (O), dây cung AB bất kì và một điểm D nằm trên (O) khác A và B.

a) Dựng đường tròn $\omega _{1}$ và $\omega _{2}$ lần lượt tiếp xúc trong, tiếp xúc ngoài với (O) và tiếp xúc AB.

b) Kẻ dây cung DE cắt đoạn AB tại C. Dựng đường tròn $\omega _{3}$ tiếp xúc với CD, CA và tiếp xúc trong với (O). 




#722749 Chứng minh rằng FG, EH và MN đồng quy.

Đã gửi bởi toanND on 05-06-2019 - 09:39 trong Hình học

Cho tam giác ABC, các điểm E, F thuộc CA, AB sao cho B, C, E, F cùng nằm trên một đường tròn. Đường tròn (B, BE) cắt CA tại G khác E. Đường tròn (C, CF) cắt AB tại H khác F. Đường tròn (B, BE) cắt (C, CF) tại M, N. Chứng minh rằng FG, EH và MN đồng quy.

Capture 5.PNG




#722758 Hướng vẽ hình bằng Geogebra

Đã gửi bởi toanND on 05-06-2019 - 10:11 trong Thử các chức năng của diễn đàn

[attachment=Capture 6.png]




#722762 CM: B,O,D thẳng hàng

Đã gửi bởi toanND on 05-06-2019 - 10:28 trong Hình học

Từ câu b ta có $\frac{AB}{BH}=\frac{EK}{OK}\Rightarrow \frac{AB}{2BH}=\frac{EK}{2OK}\Leftrightarrow \frac{AB}{BC}=\frac{EK}{AK}$

Xét hai tam giác ABC và EKA có $\widehat{ABC}=\widehat{AKE}; \frac{AB}{BC}=\frac{EK}{AK}$

 nên hai tam giác đó đồng dạng theo trường hợp c.g.c

suy ra $\widehat{EAK}=\widehat{ACB}=\widehat{AKB}\Rightarrow AE|| BK\Rightarrow AE\perp AB$.

Từ đó dễ dàng suy ra đpcm

Capture 6.PNG




#722765 Chứng minh BK chia đôi MF.

Đã gửi bởi toanND on 05-06-2019 - 11:11 trong Hình học

Capture 7.PNG

dễ thấy E là điểm chính giữa cung lớn BC nên E, O, M thẳng hàng và $EM\perp BC$.

Gọi I là giao điểm của FM với AC. Dễ thấy FM là đường thẳng Simson của tam giác ABC nên $EI \perp AC$.

Lại có $\widehat{FAE}=\widehat{FEI}\Rightarrow \Delta AEF=\Delta AEI\Rightarrow AF=AI,EF=EI$

do đó AE là trung trực của FI $\Rightarrow AK\perp FM$. (1)

MN là đường trung bình của tam giác ABC$\Rightarrow MN||AB\Rightarrow MK\perp EF$ (2)

Từ (1) và (2) suy ra E là trực tâm của tam giác MFK $\Rightarrow EM\perp FK\Rightarrow FK||BM$

Suy ra FKMB là hình bình hành $\Rightarrow đpcm$




#722771 Dựng đường tròn $\omega _{1}$

Đã gửi bởi toanND on 05-06-2019 - 17:22 trong Hình học

 

b) Cách dựng $\omega_{3}:$ Gọi tâm nội tiếp $\Delta DAE$ là $L,O_3$ là hình chiếu $L$ lên phân giác trong $\widehat{ACD}.$ Khi đó đường tròn tâm $O_3$ tiếp xúc $CA$ chính là $\omega_{3}$ (bổ đề Sawayama-Thebault).

sao k đúng nhỉ?

@halloffame: Mình chỉnh lại rồi nhé.




#722787 CM HM vuông AK

Đã gửi bởi toanND on 05-06-2019 - 23:06 trong Hình học

N là điểm gì vậy?




#722800 Chứng minh bất đẳng thức

Đã gửi bởi toanND on 06-06-2019 - 16:59 trong Bất đẳng thức và cực trị

$a+bc = (a+b+c)a+bc= (a+b)(a+c)$

$\Rightarrow \sum \frac{bc}{\sqrt{a+bc}}=\sum \frac{bc}{\sqrt{(a+b)(b+c)}}\leq \frac{1}{2}\sum (\frac{bc}{a+b}+\frac{bc}{a+c})=\frac{1}{2}\sum (\frac{bc}{a+b}+\frac{ca}{a+b})=\frac{1}{2}(a+b+c)=\frac{1}{2}$




#722861 chứng minh bất đẳng thức

Đã gửi bởi toanND on 08-06-2019 - 16:22 trong Bất đẳng thức và cực trị

Ta đi chứng minh BĐT sau : $\frac{a}{a^{2}+1}\leq \frac{18}{25}a+\frac{3}{50}$ với $a\geq 0$.

Sau khi thu gọn thì bất đẳng thức trên tương đương với $(3a-1)^{2}(4a+3)\geq 0$ luôn đúng với mọi a không âm.

Làm tương tự với các BĐT còn lại rồi cộng lại ta được đpcm. 

Dấu đẳng thức xảy ra khi $a=b=c=\frac{1}{3}$




#722862 BDT

Đã gửi bởi toanND on 08-06-2019 - 16:49 trong Bất đẳng thức và cực trị

mình nghĩ là có đk a,b,c không âm




#722873 chứng minh bất đẳng thức

Đã gửi bởi toanND on 09-06-2019 - 08:25 trong Bất đẳng thức và cực trị

Cách a cũng giống 

Sin99